Fake proof that $\frac{e^x-1}{e^x+1}=e^x$, via integrating $\operatorname{sech} x$ in two ways

We start with the integral:
$$\int \text{sech}(x)dx$$


Method 1
\begin{align} \int \text{sech}(x)dx & = \int\frac{2}{e^x+e^{-x}}dx \\ &= \int\frac{2e^x}{e^{2x}+1}dx \end{align} Using the substitution $u=e^x$,
\begin{align} \int \text{sech}(x)dx & = \int\frac{2}{u^2+1}du \\ &= 2\text{ arctan}(u)+c \\ &= 2\text{ arctan}(e^x)+c \end{align}


Method 2
\begin{align} \int \text{sech}(x)dx & = \int\frac{\text{cosh}(x)}{\text{cosh}^2(x)}dx \\ & = \int\frac{\text{cosh}(x)}{\text{sinh}^2(x)+1}dx \end{align} Using the substitution $u=\text{sinh}(x)$, \begin{align} \int \text{sech}(x)dx & = \int\frac{1}{u^2+1}du \\ &= \text{arctan}(\text{sinh}(x))+c \\ &= 2\text{ arctan}(\text{tanh}(\frac{x}{2}))+c \\ &= 2\text{ arctan}(\frac{e^x-1}{e^x+1})+c \end{align}


Thus, we obtain:

$$\frac{e^x-1}{e^x+1}=e^x$$

However, I see no reason why they are equal. Did I do something wrong in the calculation?


Solution 1:

Because $2\arctan\left(\frac{e^x-1}{e^x+1}\right)=2\left(\arctan(e^x)-\frac\pi4\right)=2\arctan(e^x)+C'$.

The results differ by a constant.